Not A?
Hey, I'm just a little confused why answer A wouldn't weaken the argument. If most of them aren'...
ttaylor1207 on May 18, 2020
  • December 2004 LSAT
  • SEC2
  • Q17
5
Replies
C and E
Hello, when choosing I was torn between C and E and went E on time. Would you be able to just exp...
Jermaine1 on May 17, 2020
  • December 2004 LSAT
  • SEC2
  • Q10
1
Reply
Answer Choices are out of order
I think the answer choices are out of order. For me, C says The ozone layer regenerates at a slow...
tomgbean on May 14, 2020
  • December 2004 LSAT
  • SEC2
  • Q2
4
Replies
Can you explain why A and C are not the correct...
"But that does not mean that the government's view is necessarily the correct view; anyone who th...
monicanimm on April 21, 2020
  • December 2004 LSAT
  • SEC2
  • Q16
1
Reply
Please explain why A isn't a necessary assumption
Please explain why A isn't a necessary assumption for the argument. Thanks!
amf on March 10, 2020
  • December 2004 LSAT
  • SEC2
  • Q3
1
Reply
Cause and Effect Relationship
I am having trouble identifying the cause and effect relationship we are trying to weaken. I unde...
EmilyMarieMenendez on March 10, 2020
  • December 2004 LSAT
  • SEC2
  • Q11
3
Replies
How to Think About Weaken Questions
Hi, I've read the answers to people's questions and I think I understand yalls explanations (sort...
odsimkins on March 1, 2020
  • December 2004 LSAT
  • SEC2
  • Q11
2
Replies
I don't understand this question.
Can someone please give a thorough explanation?
nimakian801 on February 7, 2020
  • December 2004 LSAT
  • SEC2
  • Q19
3
Replies
Is there a typo on the answer choices and respo...
Below is what I have for my answer choices and they are not matching up to the explanations: A...
#JW on February 1, 2020
  • December 2004 LSAT
  • SEC2
  • Q6
1
Reply
Terminology
This question is very hard :( I think I consistently get these types of questions wrong because I...
hannahjoannelin on January 30, 2020
  • December 2004 LSAT
  • SEC2
  • Q19
1
Reply
Identifying Conclusion
I mistakenly associated the first sentence as the conclusion. Could you please explain why the se...
mahosmar on January 20, 2020
  • December 2004 LSAT
  • SEC2
  • Q16
1
Reply
Question Type?
Would this be a MBT question?
hkcollard53 on January 15, 2020
  • December 2004 LSAT
  • SEC2
  • Q8
1
Reply
Diagramming assistance
I've diagrammed this statement as PS - >AM or R. The contrapositive of the statement being Not AM...
awashing on December 24, 2019
  • December 2004 LSAT
  • SEC2
  • Q12
2
Replies
Difference between C and D
Could you explain how C is different from D, as Maria pointed that Popular music failed to repres...
thom on November 27, 2019
  • December 2004 LSAT
  • SEC2
  • Q5
2
Replies
Diagram
How would you recommend approach this question? Would diagramming be helpful in this kind of ...
thom on November 27, 2019
  • December 2004 LSAT
  • SEC2
  • Q18
1
Reply
Answer Explanation
Why could C not be correct if the factors are "uncontrollable"
JennaArthur on November 11, 2019
  • December 2004 LSAT
  • SEC2
  • Q21
1
Reply
C and D as sufficient?
Would you say that answer choices C and D are incorrect because they are sufficient conditions?
shafieiava on November 5, 2019
  • December 2004 LSAT
  • SEC2
  • Q3
1
Reply
Mismatched question options
Hello, I've noticed on this question and another that the questions options are mismatched. Th...
b_theo on October 27, 2019
  • December 2004 LSAT
  • SEC2
  • Q17
2
Replies
Technical Difficulties
the diagnostic test got stuck in question number 3
icsparalegal on October 16, 2019
  • December 2004 LSAT
  • SEC2
  • Q3
1
Reply
Question Type ?
What is the question type for this type of question?
riaunna-bowie on October 15, 2019
  • December 2004 LSAT
  • SEC2
  • Q21
1
Reply